Đến nội dung

wtuan159 nội dung

Có 279 mục bởi wtuan159 (Tìm giới hạn từ 26-04-2020)



Sắp theo                Sắp xếp  

#556456 Tính tích phân $\int \frac{dx}{xln(x)}$

Đã gửi bởi wtuan159 on 26-04-2015 - 18:09 trong Tích phân - Nguyên hàm

$\int \frac{dx}{xln(x)}$

 

$\int \frac{ln(x)}{dx}$




#530179 Tìm n để tổng sau là số chính phương.

Đã gửi bởi wtuan159 on 23-10-2014 - 19:51 trong Số học

Tìm số tự nhiên n khác 0 sao cho tổng $1! + 2! + 3! + ... + n!$ là một số chính phương.

đặt s(n) = 1! + 2! + ... + n! 
s(1) = 1 và s(3) = 9 là số chính phương. 
s(2) = 3 và s(4) = 33 không là số chính phương. 
Với n ≥ 5 có n! chia hết cho 10 - do trong tích có 2 thừa số là 2 và 5 - nên n! tận cùng bằng 0 
Vậy với n ≥ 5 có s(n) = s(4) + 5! + ... + n! tận cùng bằng 3. Do số chính phương không tận cùng bằng 3 (chỉ tận cùng bằng 0, 1, 4, 5, 6, 9) nên với n ≥ 5 có s(n) không là số chính phương. 
Vậy chỉ với n = 1 và n = 3 tổng đã cho là số chính phương.




#525331 tài liệu toán lớp 10

Đã gửi bởi wtuan159 on 20-09-2014 - 12:28 trong Tài liệu tham khảo khác

mọi người ai có tài liệu gì của toán lớp 10 nâng cao ko ạ, cả đại cả hình ấy ạ cho mình tham khảo với. Thanks Thanks :icon6:  @};-  :)

Chỉ bạn website chuyên video ôn thi từ lớp 10 đến lớp 12 miễn phí

 

http://huongnghiep.vn/On-thi_c26.htm




#525049 $3x^2-2^{log_2x^3+1}=log_2(x^2+1)-log_2x$

Đã gửi bởi wtuan159 on 18-09-2014 - 09:40 trong Phương trình - hệ phương trình - bất phương trình

TH t<0 thì VP(*) chưa chắc âm . VD t= -0,5 thì VP = 2,5 >0

TH t>0 $f'(t)=-3.2^{2-t^2(2t+3)}tlog_2(t+1)+\frac{t(t+2)+2}{(t+1)^2}-2$ thì c chứng minh $f'(t)<0$ thế nào

 Ừ mình kiểm tra lại rồi. Cách mình sai rồi. Mình cũng ko chắc chắn 100% ngay từ đầu. Ai giúp bạn ấy đi nhé




#524798 $\sqrt{4x^{2}-5x+6}+\sqrt{x^{2...

Đã gửi bởi wtuan159 on 16-09-2014 - 09:48 trong Phương trình - hệ phương trình - bất phương trình

Giải PT:$\sqrt{4x^{2}-5x+6}+\sqrt{x^{2}-3x+8}= \sqrt{4x^{2}-7x+8}+\sqrt{x^{2}-8x+11}+\sqrt{6}-2$

Sorry các bạn nhé !PT đúng phải là ở đây!

Lấy máy tính bấm thử ra nghiệm $x=1$ rồi đấy bạn dùng lương liên hợp mà đặt thừa số chung :)




#524794 $\sqrt{1+x^2}(\sqrt{1-x^2}+\sqrt...

Đã gửi bởi wtuan159 on 16-09-2014 - 09:22 trong Phương trình, hệ phương trình và bất phương trình

giải bất phương trình $\sqrt{1+x^2}(\sqrt{1-x^2}+\sqrt{2x})\leq 1-2x-x^2$

Bài này có nghiệm hay đấy

 

Đk pt : $0\leq x\leq 1$

Đặt $\left\{\begin{matrix} a=\sqrt{2x} \\ b=\sqrt{1-x^{2}} \end{matrix}\right.$

 $a\geq 0,b\geq 0$

BPT <=> $\sqrt{2-b^{2}}(b+a)\leq b^{2}-a^{2}$

Do $(b+a)$ luôn > 0 với x thuộc đoạn $[0;1]$ > nên ta có quyền chia cả 2 về cho $(b+a)$ mà BPT ko đổi dấu

 

=> $\sqrt{2-b^{2}}\leq (b-a)$ (*)

Thay a,b vào lại 

(*) <=> $\sqrt{2x}\leq \sqrt{1-x^{2}}-\sqrt{x^{2}+1}$

Đến đây giải bình thường 

<=>$\sqrt{2x}+\sqrt{1+x^{2}}\leq \sqrt{1-x^{2}}$

<=>$2x^{2}+2x+2\sqrt{2x^{3}+2x}\leq 0$

<=>$\sqrt{2x^{3}+2x}\leq -(x^{2}+x)$

$\left\{\begin{matrix} -1\leq x\leq 0\\ x(x^{3}+x-2)\geq 0 \end{matrix}\right.$

<=>$\left\{\begin{matrix} -1\leq x\leq 0\\ \begin{bmatrix} x\leq 0\\ x\geq 1 \end{bmatrix} \end{matrix}\right.$

So sánh điều kiện ban đầu kết luận BPT có nghiệm duy nhất $x=0$  ~O)




#524632 $3x^2-2^{log_2x^3+1}=log_2(x^2+1)-log_2x$

Đã gửi bởi wtuan159 on 15-09-2014 - 15:04 trong Phương trình - hệ phương trình - bất phương trình

Giải phương trình:

$$3x^2-2^{log_2x^3+1}=log_2(x^2+1)-log_2x$$

ĐK : x>0

PT <=> $3x^{2}-2x^{3}=log_{2}(x^{2}+1)-log_{2}x$

$<=>(x-1)(-2x^{2}+x+1)+1=log_{2}(x^{2}-1+2)-log_{2}(x-1+1)$

$<=>(x-1)^{2}(-2x-1)+1=log_{2}(\frac{x^{2}-1+2}{x-1+1})$

$<=>2^{1+(x-1)^{2}(-2x-1)}=\frac{(x-1)(x-1+2)+2}{x-1+1}$

 

Đặt $t=x-1$ 

PT $<=>2^{1+t^{2}(-2t-3)}=\frac{t(t+2)+2}{t+1}$ (*)

Gỉa sử t=0 . Ta thấy thỏa pt (*) => t=0 => x=1 (thỏa đk)

Gỉa sử t<0 . Ta thấy vế trái pt (*) luôn > 0 còn vế phải (*) luôn < 0 ( TH này ko có nghiệm nào thỏa)

GIả sử t>0 . Xét $f(t)=2^{1+t^{2}(-2t-3)}-\frac{t(t+2)+2}{t+1}(*)$ trên đoạn (0;+$\infty$)

tính đạo hàm thì thấy pt luôn nghịch biến từ đó kết luận pt có nghiệm duy nhất x=1. 




#523877 $(3x-2)^{3x-5} = (\frac{1}{9x^{2}-6x+4})^{5}$

Đã gửi bởi wtuan159 on 11-09-2014 - 11:21 trong Phương trình - hệ phương trình - bất phương trình

1. $(3x-2)^{3x-5} = (\frac{1}{9x^{2}-6x+4})^{5}$

2, $4^{x} -3.6^{x}+2.9^{x}=0$

Mình mới học phần này nhưng k hiểu, các bạn giúp mình với, nếu có thể thì nói giảng kỹ để mình hiểu nhé. Tks nhiều

1/ 

 

Pt $<=>(3x-2)^{3x-5}=(9x^{2}-6x+4)^{-5}$

 

  $<=>(3x-2)^{3x-5}=[(3x-2)(3x-2)+2(3x-2)+4]^{-5}$

  $<=>(3x-2)^{3x}=[(3x-2)+2+\frac{4}{3x-2}]^{-5}$  ( Vì $x=\frac{2}{3}$ ko phải nghiệm)

 

$<=>(3x-2)^{3x}=(3x+\frac{4}{3x-2})^{-5}$

 

Đặt $\left\{\begin{matrix} u=3x\\v=3x-2 =>u=v+2 \end{matrix}\right.$

Ta có được hệ 

 

$\left\{\begin{matrix} v^{u}=(u+\frac{4}{v}(1))^{-5}\\ u=v+2(2) \end{matrix}\right.$

 

Thế (2) vào (1)

$=>v^{v+2}=(v+2+\frac{4}{v})^{-5}$ (*)

Đến đây bạn lấy máy tính bấm thử thì thấy pt vô nghiệm

 

Vậy bạn chỉ cần CM (*) vô nghiệm là được rồi .

 

2/ Bài 2 thì cơ bản rồi. Dạng này thì bạn có thể chọn 1 trong 3 số  $4^{x},6^{x},9^{x}$ rồi lấy pt chia cho 1 trong 3 số ấy

Ở đây mình chọn $9^x$

Pt sau khi lấy các số hạng chia cho $9^x$

 

<=>$(\frac{2}{3})^{2x}-3.(\frac{2}{3})^{x}+2=0$

Đến đây đặt t giải pt bậc 2 là xong .Đk t >0 nhé




#523776 Tìm k thỏa mãn: $(k+50)x^2-2x+\frac{1}{100}...

Đã gửi bởi wtuan159 on 10-09-2014 - 18:08 trong Phương trình - hệ phương trình - bất phương trình

CT trên đúng rồi . Lý thuyết có mà bạn. Đáp án k $\geq 50$  . Bài 2 làm tương tự 




#523641 Tìm m để tiếp tuyến với (C) tại giao điểm của (C) với Oy tạo với 2 trục tọa đ...

Đã gửi bởi wtuan159 on 09-09-2014 - 17:02 trong Hàm số - Đạo hàm

Cho hàm số y=$x^{3}$-mx+1-m.(C).Tìm m để tiếp tuyến với (C) tại giao điểm của (C) với Oy tạo với 2 trục tọa độ 1 tam giác có diện tích =8

$(C)\cap (Oy)$=>$ x_{0}=0,y_{0}=1-m$

 

$A(0,1-m)$

 

$f'(x)=3x^{2}-m$

 

=>$f'(0)=-m$

PTTT với (C) tại $A(0,1-m)$:

 

$y=-mx+1-m(*)$

 

$(*)$ giao với trục 0x <=> $-mx+1-m=0$ =>$x=\frac{1-m}{m}$  $B[\frac{1-m}{m},0)$;

$(*)$ giao với trục 0y => $x=0,y=1-m$   $C(0,1-m)$

 

$S_{OBC}=\frac{1}{2}.\left | [\vec{OB},\vec{OC}] \right |$

 

<=> $16=\left | \frac{(1-m)^{2}}{m} \right |=\frac{(1-m)^{2}}{\left | m \right |}$

<=>$16\left | m \right |=(1-m)^{2}$

<=>$\begin{bmatrix} 16m=m^{2}-2m+1\\ -16m=m^{2}-2m+1 \end{bmatrix}$

 

Qua đó tìm được các điểm $\begin{bmatrix} m=9\pm 4\sqrt{5}\\ m=-7\pm 4\sqrt{3} \end{bmatrix}$   ~O)




#523245 $\sqrt[4]{x}+\sqrt[4]{x-1}=\sqrt[4]...

Đã gửi bởi wtuan159 on 07-09-2014 - 11:26 trong Phương trình, hệ phương trình và bất phương trình

8$3\sqrt[3]{x^3+8}=2x^2-3x+10$
9.$\sqrt{x^2-x-6}+x^2-x-18=0$
10.$3x^2-x+3=\sqrt{3x+1}+\sqrt{5x+4}$
11.$2x^3-x^2+\sqrt[3]{2x^3-3x+1}=3x+1+\sqrt[3]{x^2+2}$

Bài 11 nhìn kỹ là rất đẹp bạn à.

 

Biến đổi pt 1 tí là thấy 

 

pt <=> $2x^{3}-3x+1+ \sqrt[3]{2x^{3}-3x+1}=x^{2}+2 + \sqrt[3]{x^{2}+2}$

   Đặt a=$\sqrt[3]{2x^{3}-3x+1}$ => $a^{3}=2x^{3}-3x+1$

          b= $\sqrt[3]{x^{2}+2}$ =>$b^{3}=x^{2}+2$

 

Thế a,b vào pt :

pt  <=> $a^{3}+a=b^{3}+b <=> (a-b)(a^{2}+ab+b^{2})+(a-b)=0$

    <=>$(a-b)(a^{2}+ab+b^{2}+1)=0$

   

   <=> $\begin{bmatrix} a=b (1)\\ (a+\frac{1}{2}b)^{2}+(\frac{3}{4}b^{2}+1)=0 (2) \end{bmatrix}$

 

Nhận thấy pt (2) luôn >0 với mọi a,b thuộc R

 

Vậy giải pt a=b quá đơn giản rồi   ~O)

 

Gợi ý giải pt a=b có chia hooc- ne . 




#515569 Giải phương trình AX=B đối với ẩn là ma trận X

Đã gửi bởi wtuan159 on 26-07-2014 - 17:09 trong Đại số tuyến tính, Hình học giải tích

với $A=\begin{bmatrix} 1 & -1 &1 \\ -1 &2 &1 \\ -2&3 &1 \end{bmatrix}$ và  $B=\begin{bmatrix} 1 &1 &1 &-1 \\ 1& 0 &2 &2 \\ 1& -2 & 2 & 0 \end{bmatrix}$




#513244 Tìm điều kiện của tham số m để hàm số đơn điệu trên một khoảng bằng phương ph...

Đã gửi bởi wtuan159 on 16-07-2014 - 19:44 trong Chuyên đề toán THPT

cho em hỏi các bạn và  thầy bài này 1 chút

tìm m để y=x3-6mx2+2(12m-5)x+1 đồng biến trên (-âm vô cực,0) hợp với (3,+dương vô cực)

bài này thì phải xét 2 trường hợp ạ.

th1 : x<0

và th2: x>3

mong m.n chỉ bảo

Bạn tìm y', tính denta. Sau đó xét 3 TH denta=0, delta<0,delta> 0 . Rồi xét từ từ. Chung quy nó là vậy 




#513089 Cm $\begin{vmatrix} 2 & 0 & 4\\ 5...

Đã gửi bởi wtuan159 on 16-07-2014 - 07:23 trong Đại số tuyến tính, Hình học giải tích

Phần $100.c_{1}+10.c_{2}+c_{3}$ là sao mình chưa hiểu? Làm vậy định thức ko thay đổi à




#513088 $x^{2}-3x+1=-\frac{\sqrt{3}}...

Đã gửi bởi wtuan159 on 16-07-2014 - 07:19 trong Phương trình, hệ phương trình và bất phương trình

phương trình tương đương $-(x^2+x+1)+2(x^2-x+1)=\frac{-\sqrt{3}}{3}\sqrt{(x^2+x+1)(x^2-x+1)}$

tới đây được dạng đẳng cấp rồi

Làm rõ thêm đi bạn




#512977 Cm $\begin{vmatrix} 2 & 0 & 4\\ 5...

Đã gửi bởi wtuan159 on 15-07-2014 - 17:46 trong Đại số tuyến tính, Hình học giải tích

Biết rằng các số 204,527,255 chia hết cho 17 .Hãy chứng minh $\begin{vmatrix} 2 & 0 & 4\\ 5 &2 & 7\\2 &5 &5 \end{vmatrix}$ chia hết cho 17




#512976 Tìm m để hàm số $y=mx+(m-1)x^2+2\sqrt{x^2-1}$ nhận t...

Đã gửi bởi wtuan159 on 15-07-2014 - 17:37 trong Hàm số - Đạo hàm

Đk: $\left\{\begin{matrix} x\leq -1\\ x\geq 1 \end{matrix}\right.$

Hàm số nhận trực Oy làm trục đối xứng khi và chỉ khi hàm số là hàm số chẵn

Khi đó: $f(x)=f(-x)$

<=>$mx+(m-1)x^{2}+2\sqrt{x^{2}-1}=m.(-x)+(m-1).(-x)^{2}+2\sqrt{(-x)^{2}-1}$

<=>$2mx$=0

<=>$m=0$ Do x khác 0

Vậy $m=0$ là giá trị cần tìm




#512098 $A^{2}-3A+I=0$ thì $A^{-1}=3I-A$

Đã gửi bởi wtuan159 on 10-07-2014 - 17:19 trong Đại số tuyến tính, Hình học giải tích

Nghĩa là chứng tỏ A khả đảo đúng ko bạn?  :)




#512083 $A^{2}-3A+I=0$ thì $A^{-1}=3I-A$

Đã gửi bởi wtuan159 on 10-07-2014 - 16:38 trong Đại số tuyến tính, Hình học giải tích

1/CMR nếu A là ma trận vuông thỏa $A^{2}-3A+I=0$ thì $A^{-1}=3I-A$

2/ Cm:$\begin{vmatrix} b+c &c+a &a+b \\ b'+c'&c'+a' &a'+b' \\b''+c'' &c''+a'' &a''+b'' \end{vmatrix}=2.\begin{vmatrix} a &b &c \\a' &b' &c' \\a'' &b'' &c'' \end{vmatrix}$




#511957 $\left\{\begin{matrix} x^{2}+2...

Đã gửi bởi wtuan159 on 09-07-2014 - 21:32 trong Phương trình - hệ phương trình - bất phương trình

Giải phương trình:

$\left\{\begin{matrix} x^{2}+2\sqrt{x^{2}+4x+16}=4y & & \\ x^{2}+y^{2}-xy=4 & & \end{matrix}\right.$

Tìm được mỗi 1 nghiệm (0;2) hình như còn nghiệm mà ko làm ra.Ai giúp thêm với :)




#511646 Tìm điểm trên trục hoành để từ đó kẻ được hai tiếp tuyến với $(C ):...

Đã gửi bởi wtuan159 on 08-07-2014 - 12:52 trong Hàm số - Đạo hàm

Cho hàm số $(C ):\,\,\, y=x^3-3x^2+4$
Tìm điểm trên trục hoành để từ đó kẻ được hai tiếp tuyến với $(C )$ và hai tiếp tuyến này vuông góc với nhau.
Cảm ơn vì sự giúp đỡ.

 
y'=$3x^{2}-6x$
Gọi $A(a;0)$ thuộc Ox
Pt tiếp tuyến qua A và có hệ số góc k :$y=k(x-a)$
Ta có hệ :
$\left\{\begin{matrix} x^{3}-3x^{2}+4=k(x-a)(1)\\ 3x^{2}-6x=k(2) \end{matrix}\right.$
Thế (2) vào (1)
=>$x^{3}-3x^{2}+4=3x^{3}-6x^{2}-3ax^{2}$<=>$2x^{3}-3(a+1)x^{2}+6ax-4=0 <=>(x-2)[2x^{2}-(3a-1)x+2]=0$
pt luôn có 1 nghiệm x=2 mà f'(2)=0 => tiếp tuyến song song trục hoành => Ko có tiếp tuyến nào của (C) vuông góc với tiếp tuyến này.Do đó YCBT<=> tìm a để pt $2x^{2}-(3a-1)x+2=0(*)$ phải có 2 nghiệm p/b thỏa $(3x_{1}^{2}-6x_{1}).(3x_{2}^{2}-6x_{2})=-1$ và (a<-1,a>$\frac{5}{3})$
<=>$9(x_{1}x_{2})^{2}-6(x_{1}+x_{2})+36x_{1}x_{2}=-1$ 
Theo Viet từ (*) thế vào và ra kết quả.Nhớ so sánh đk của a



#511315 Cho tam giác ABC, Tìm A?

Đã gửi bởi wtuan159 on 06-07-2014 - 21:00 trong Phương pháp tọa độ trong mặt phẳng

... :D

cái lí gì thế bạn :), đề đâu có cho, nói như bạn, cứ giả sử F nằm ở bất cứ vị trí nào đó trên ED thì cũng thỏa mãn dữ kiện đề bài, chẳng lẽ chạy theo từng vị trí để giải à, hì :D

ừ mình cũng nghi ngờ mà hihi. Bạn giải bài này thử xem. Cách giải kia mình ko hiểu




#511313 Cho tam giác ABC, Tìm A?

Đã gửi bởi wtuan159 on 06-07-2014 - 20:58 trong Phương pháp tọa độ trong mặt phẳng

:)




#511294 Cho tam giác ABC, Tìm A?

Đã gửi bởi wtuan159 on 06-07-2014 - 20:29 trong Phương pháp tọa độ trong mặt phẳng

Điều quan trọng là đừng nên ngộ nhận không thì hỏng bét =)

Mình rất tiếc nhưng sự thật là thế, lần sau cẩn thận bạn nhá, kể ra ngồi gõ được thế cũng toát mồ hôi hột, nhờ  :icon6:

Mình thấy đáp án hợp lý mà.Mình cũng nghi nghi chỗ màu đỏ từ trước nhưng F trên ED là hình chiếu của H cũng được mà.Nó thỏa dữ kiện đề bài




#511290 $\left\{\begin{matrix}27x^3y^3+7y^3=8...

Đã gửi bởi wtuan159 on 06-07-2014 - 20:23 trong Phương trình - hệ phương trình - bất phương trình

9)$$\left\{\begin{matrix}2y-x-m=0 (1)\\x+\sqrt{xy}=1(2)\end{matrix}\right.$$

Từ (1) =>$y=\frac{m+x}{2}$

Thế $y=\frac{m+x}{2}$ vào (2)

<=>$x+\sqrt{\frac{mx+x^{2}}{2}}=1<=>\sqrt{\frac{mx+x^{2}}{2}}=1-x$

ĐK:x$\leq$1

Bình phương 2 vế ra pt bậc 2:$\frac{1}{2}x^{2}-(2+\frac{1}{2}m)x+1=0 (*)$

 Hệ pt có 1 nghiệm duy nhất khi và chỉ khi pt(*) có 1 nghiệm duy nhất $\leq 1$

Khi đó $\Delta =0<=>m=-4\pm 2\sqrt{2}$